winer profrespons fall 2010

47
Professional Responsibility Fall 2010 Laurence Winer I. Sources of Ethical Rules A. The law trumps everything else! 1. Constitutional Law a. 1st Amendment - restrictions of lawyer speech i. Prohibitions: (1) Lawyers are restricted from giving certain kinds of advice even if not unlawful. (2) Lawyers are restricted in making public comments about pending cases (3) Lawyers may not make extrajudicial comments about judges generally or the administration of justice Gentile v. Nevada State Bar: because lawyers are participants in a dignified system and owe a fiduciary duty not to enga ge in public debate that would be detrimental to either the participants in a trial or the justice system itself, lawyer's first amendment rights must be restricted. (i) Model Rules: 3.6(c) -> a lawyer may make some disclosures to the press. It must be a statement that a "reasonable lawyer would believe is required to  protect a client from substantial undue prejudice." 8.2(a) -> a lawyer shall not make criticisms of a judge by making false statements. (4) Lawyers may not publicly criticize a judge ii. Restrictions on lawyerly advertising (1) Central Hudson test for legal advertising by lawyers: Regulation of advertising acceptable when: (a) Government interest is substantial (b) Regulation directly addresses interest (c) Regulation is narrowly tailored to serve the interest (2) Model Rules:

Upload: justisiusmaximus

Post on 06-Apr-2018

219 views

Category:

Documents


0 download

TRANSCRIPT

Page 1: Winer Profrespons Fall 2010

8/2/2019 Winer Profrespons Fall 2010

http://slidepdf.com/reader/full/winer-profrespons-fall-2010 1/47

Professional Responsibility Fall 2010

Laurence Winer 

I. Sources of Ethical Rules

A. The law trumps everything else!

1. Constitutional Law

a. 1st Amendment - restrictions of lawyer speech

i. Prohibitions:

(1) Lawyers are restricted from giving certain kinds of advice even if not

unlawful.

(2) Lawyers are restricted in making public comments about pending cases

(3) Lawyers may not make extrajudicial comments about judges generally or 

the administration of justice

Gentile v. Nevada State Bar: because lawyers are participants in a

dignified system and owe a fiduciary duty not to engage in public debate

that would be detrimental to either the participants in a trial or the justice

system itself, lawyer's first amendment rights must be restricted.

(i) Model Rules:

3.6(c) -> a lawyer may make some disclosures to the press. It must be

a statement that a "reasonable lawyer would believe is required to protect a client from substantial undue prejudice."

8.2(a) -> a lawyer shall not make criticisms of a judge by making false

statements.

(4) Lawyers may not publicly criticize a judge

ii. Restrictions on lawyerly advertising

(1) Central Hudson test for legal advertising by lawyers: Regulation of 

advertising acceptable when:

(a) Government interest is substantial

(b) Regulation directly addresses interest

(c) Regulation is narrowly tailored to serve the interest

(2) Model Rules:

Page 2: Winer Profrespons Fall 2010

8/2/2019 Winer Profrespons Fall 2010

http://slidepdf.com/reader/full/winer-profrespons-fall-2010 2/47

7.2(a) -> a lawyer may advertise but advertisements must include the

name and office address of at least one lawyer or law firm, advertising

must be truthful and not misleading, and lawyers are restricted in how they

advertise and the type of content each advertisement contains.

 b. 5th Amendment - protections against self incrimination by lawyers

i. Spevack v. Klein: two principle propositions flow from this case:

(1) A lawyer cannot be disciplined for failure to respond to a judicial

 proceeding when the failure consists of invoking a constitutional privilege.

(2) The Fifth Amendment can be used in a disciplinary hearing to avoid

incrimination.

ii. Fisher v. United States: stands for the proposition that once a lawyer is in

 possession of physical evidence, the protections of the fifth amendment are

limited.

2. Criminal Law (see section IV)

a. Fraud

 b. Aiding and Abetting

c. Perjury

d. Obstruction of Justice

3. ABA Model Rules of Professional Conduct

a. Applicable rules (model rules)

 b. Positive law (common law, statutory regulations)

c. Moral obligation

i. To act as a friend to our client

(1). Professor Charles Friend: while lawyer are expected to act as an advocate

for their clients, lawyers must also act with personal integrity exhibiting

respect to their clients and refrain from cheating, lying, or humiliating.

ii. Bad man perspective: in seeing the law through the eyes of a bad man

(avoiding the full force of the law), lawyers will have an insight into knowing the

law and effectively advocating the interests of their clients.

B. Rules on Lawyerly Billing

1. Rule 1.5 - fees

Page 3: Winer Profrespons Fall 2010

8/2/2019 Winer Profrespons Fall 2010

http://slidepdf.com/reader/full/winer-profrespons-fall-2010 3/47

2. Rule 5.6 - Restrictions on the right to practice: a lawyer shall not be party to or 

 participate in a partnership or employment agreement that restricts the right of a

lawyer to practice after the termination of the relationship.

C. Multidisciplinary and Multi-Jurisdictional Practices

1. Form of practice restrictions under Rule 5.4: 5.4 forbids a lawyer from sharing legal

fees with a nonlawyer. The rule also forbids lawyers from forming a partnership with a

non-lawyer.

a. The rationale for limiting the disciplines that can be combined in a practice are that

to protect the professional independence of judgment.

2. Responsibilities regarding law-related services - Rule 5.7: this rule provides a useful

distinction between lawyers and other kinds of professionals. Important about this rule:

clients must understand the relationship between lawyers and non-lawyers. The burden is

on the lawyer to make sure that he has taken the right measures to communicate and have

the client understand what the client needs to know and understand.

3. Multijurisdictional practice of law - Rule 5.5

D. Lawyers and the Authorized Practice of Law

II. Adversarial Ethics

A. Spaulding v. Zimmerman: a lawyer for the defendant had a doctor's report on the plaintiff 

that indicated that the plaintiff had a significant medical condition that was likely caused by

the accident. He did not disclose this information to the plaintiff prior to settlement

negotiations.

1. Model rules implicated:

Rule 1.4 -> a lawyer must communicate to his client any information concerning the

settlement agreement. Zimmerman's lawyer perhaps had an ethical duty to alert

Zimmerman of Spaulding's condition because it could potentially have impacted the

settlement. (See comment 5: a client should have sufficient information to participate

intelligently in decisions concerning the objectives of the representation).

Rule 2.1 -> a lawyer's role as a moral counselor. A lawyer can act as a moral

counselor to his client exercising independent professional judgment. In Spaulding ,

the lawyer for the defendant had a strong moral reason to let his client know what the

Dr. report had found.

Page 4: Winer Profrespons Fall 2010

8/2/2019 Winer Profrespons Fall 2010

http://slidepdf.com/reader/full/winer-profrespons-fall-2010 4/47

Rule 3.3 -> lawyers are required to show candor to a tribunal. In Spaulding , the

lawyers for the defendant lied in court about the extent of injuries to Spaulding when

it signed off on a final settlement agreement.

Rule 4.1 -> a lawyer must be truthful in the statements he makes to others.

B. Commonwealth v. Stenhach: the lawyers and their investigator were confidentially told by

the defendant about some incriminating evidence. The lawyers sent out an investigator and

he found the evidence, but the lawyers bagged it and put it in their desk drawer. The police

officer who found it warned the lawyers about hiding the rifle butt but the lawyers cited the

attorney/client privilege.

1. Model rules implicated:

Rule 1.4 -> before embarking on a journey to uncover a piece of evidence, a lawyer 

should communicate fully to the client the implications of having the evidence turned

over.

(i) Rationale: clients have a right to know the ground rules up-front before they

 put themselves into a position where they are boxed in by the ground rules. They

must understand the limits of what lawyers can do for them ethically and

 professionally.

Rule 3.4 -> A lawyer should not conceal or destroy evidence that the other side might

 be interested in.

2. The turnover rule: a criminal defense attorney has an obligation to turn over to the

 prosecution physical evidence which comes into his possession, especially where the

evidence comes into the attorney's possession through acts of a third party who is neither 

a client of the attorney nor an agent of the attorney.

III. Dual World's Between Ethics Rules and Other Bodies of Law

A. Law between the bar and the state; Koniak 

1. Main point: the ethical rules are not the be-all-end-all for lawyers. There are other 

 bodies of law that lawyers must take account of.

2. Sub-points:

a. Although state laws and ethics rules may seldom directly conflict, there are

competing interpretations by both sides that lead to battles. This leads to

Page 5: Winer Profrespons Fall 2010

8/2/2019 Winer Profrespons Fall 2010

http://slidepdf.com/reader/full/winer-profrespons-fall-2010 5/47

circumstances by lawyers where they must resist state laws in order to meet the

 prerogatives of the ethics rules

i. Example: a lawyer's duty of confidentiality to his client but obligation not to aid

and abet a crime.

 b. In dealing with conflicts between the ethics rules and state law, courts may shun a

commitment to state law nomos (socially constructed ordering of experience) and

instead defer to norms put forth by lawyers to better understand the conflict.

B. When the Hurley-Burley's Done; Koniak 

1. Main point: continuing her point above a divergence of views and purposes between

state law and the ethics rules, Koniak argues that lawyers are trained to serve their

clients. A client's objective is to achieve some goal and the lawyer is expected to find

some ways to achieve that objective in as legal of a way as possible.

2. Sub-points:

a. State bars are very dedicated to preserving the reputation and prestige of the legal

 profession. For this reason, there is more likely to be an enforcement gap between a

state law and its actual enforcement against a lawyer because the state law is fighting

a strong-willed legal profession.

 b. Although ethics rules may prohibit a certain kind of conduct by a lawyer, there is a

disconnect between interpretation by lawyers and adherence to the rules.

c. The law of the bar should be rejected because the state's law is better. The state has

the use of force on its side while the bar's rules are often misinterpreted by lawyers.

Due to devices like the option to withdraw, the bar is perpetuating a world where

lawyers will continue to help crooked clients get away with things. Koniak argues

that there is a better way and that it should be followed.

IV. Balancing Act: Conformity with the Law

A. Criminal Law

1. False Testimony

a. U.S. v. Gellene: this is a case with an inherent conflict of interest where a law firm

undertook to represent one client in a bankruptcy action where one of the creditors

had also been represented in the past by the law firm. The firm made continual

statements to the court that there were no conflicts.

Page 6: Winer Profrespons Fall 2010

8/2/2019 Winer Profrespons Fall 2010

http://slidepdf.com/reader/full/winer-profrespons-fall-2010 6/47

i. Main ethical problem: Gellene filed an official declaration with the court

(twice actually) that he had mot made any other representations of an equity

holder or creditor of Bucyrus.

Gellene had a fiduciary duty to his clients (the debtors) to act in their best

interest. Because of his breached fiduciary duty, Gellene's sentence was

enhanced.

 b. Perjury, false swearing, subordination: any false statement under oath may

constitute perjury, as long as it was an intentional falsehood and concerned

something that was material to the proceeding in which the lie was offered.

i. Two federal statutes:

(1) 1621 - general perjury statute; prosecution must show that statement was

false and that the defendant willfully lied.

(2) 1623 - false swearing; burden of the prosecution is eased because

 prosecution doesn't have to prove that falsehood was knowingly offered under 

oath and prosecution does not need to prove which statement of two

contradictory statements is false.

2. Obstruction of Justice

a. Arthur Anderson v. U.S.: A.A. was an accountant for Enron that engaged in

document shredding prior to an SEC investigation in compliance with its own

document retention policy. Internal counsel advised other partners to adhere to the

 policy. A.A. continued shredding documents until they received formal notice that the

SEC was investigating, then it stopped the shredding. Government charged

obstruction of justice, but Supreme Court dismissed because of lack of adequate

 jury instruction.

i. Main ethical problem: although what Temple did in counseling for shredding

consistent with the company policy was not criminal, it was unethical because it

was not in accordance with showing fairness to the other side (the SEC).

ii. Model rules implicated:

(1) Rule 3.4(a) -> a lawyer is obligated to show fairness to an opposing party

and counsel. Here, Temple was obstructing another party's access to evidence

Page 7: Winer Profrespons Fall 2010

8/2/2019 Winer Profrespons Fall 2010

http://slidepdf.com/reader/full/winer-profrespons-fall-2010 7/47

 by counseling that her client continue to shred documents that *might* later 

 be pertinent to an action.

 b. Obstruction of justice and ordinary lawyering activity

i. Cintolo: although Cintolo was merely offering advice to his client when he

advised his client to refuse to testify before a grand jury (not a per se unethical

or criminal activity), his advice required the client to withhold some key

information from the jury. Therefore, the lawyer was obstructing justice.

3. Secondary and Derivative Claims

a. Aiding and Abetting

i. In general

(1) Penalty is usually equal to those available for primary violations.

(2) Mental state is more than mere knowledge; one must associate himself 

with the criminal venture.

(3) Jury is allowed to infer what the lawyer knew based on facts about the

lawyer's knowledge.

 b. Conspiracy

i. In general

(1) Tacit agreement to do something illegal in concert with others suffices to

show conspiracy even if two people act in a way that allows an inference to be

made this can demonstrate conspiracy.

c. RICO

i. Lawyers are subject to being charged as racketeers but they have some

immunity if they are acting as advisors to racketeer groups.

ii. Elements: (any of the following can constitute racketeering)

(1) Investing income derived from racketeering

(2) Acquiring an interest through racketeering pattern

(3) Participation in an enterprises affairs through racketeering.

B. Tort Law Claims by Non-Clients

1. Negligent Misrepresentation

a. Greycas v. Proud: Proud was a lawyer hired by his client, a debtor of Greycas, to

conduct a records-check on Proud's credit history in order to show that he had good

Page 8: Winer Profrespons Fall 2010

8/2/2019 Winer Profrespons Fall 2010

http://slidepdf.com/reader/full/winer-profrespons-fall-2010 8/47

credit. Proud wrote a letter to Greycas attesting to his client's financial security (when

in fact his client had several prior liens on the property.)

i. Main ethical problem: Proud did not conduct due diligence in following up on

what his client told him about his history. Due to the facts and circumstances of 

the case (standard to find due diligence practiced), Proud should not have taken

the client at his word.

ii. Negligent misrepresentation. Two elements:

(1) Privity of Contract: even though Proud had no contract with Greycas

(Proud client's creditor), Proud still had a duty to use due care to ensure that

the info he was offering was correct; this info was relied upon by a company

to do business.

(2) Contributory Negligence: Court denied that there was contributory

negligence here because we cannot require people to look out for other 

 people's negligence. Here, what Proud had to do was so simple that Greycas

would not really have an incentive to conduct a background check.

 b. Opinion Letters and Fraud

i. Model Rules Implicated:

Rule 2.3 -> evaluation for use by third persons. Lawyer can provide

evaluation of a matter affecting a client for the use by someone but only if the

client reasonably believes that the evaluation is compatible with other aspects

of the lawyer's relationship with the client.

(1) Note: ALWAYS get the client's opinion before writing any letter to be

used by the other side that may contain confidential information. The

silverado accord written by the ABA lays out the requirements for a client

opinion letter. An ideal letter contains language that says "You have asked

for our opinion on XYZ" and state what has been reviewed (the basis of 

the opinion).

2. Assisting a Client in Tortious or Illegal Conduct

a. Hazard - How far can a lawyer go in assisting clients in unlawful conduct?

i. Generally: Hazard offers this continuum between least instrumental means on

one end (offering advice) and pure instrumental means on the other (physical

Page 9: Winer Profrespons Fall 2010

8/2/2019 Winer Profrespons Fall 2010

http://slidepdf.com/reader/full/winer-profrespons-fall-2010 9/47

execution of a client's wishes by the lawyer on behalf of a client to accomplish a

 purpose the client cout not by himself accomplish).

ii. Rule 1.2 line between offering assistance and advice: 

(1) A lawyer shall not counsel or assist a client in illegal matters. Lawyers

may advise a client on particular matters.

(2) Comment 9: points out the distinction between preventing something and

suggesting how a crime could be committed.

iii. Grey areas between least instrumental and purely instrumental: the dichotomy

illuminated by 1.2(d) illustrates the grey areas that a lawyer faces in terms of his

duty to counsel a client without helping him engage in fraud or illegal activity.

iv. Practical knowledge presumption:  lawyers have the practical knowledge to

actually know when they are doing something wrong.

-> Agency law, tort law, criminal law all provide sources of guidance for what

activity is illegal.

v. Knowing when a client's actions are illegal:

(1) Client's acknowledgement of wrongdoing

(2) Lawyer's actual knowledge of the wrongdoing

(3) What did lawyer do with this knowledge when he came across it.

C. Law Firms and Multilawyer Relationships

1. In general: within law firms there is a dynamic between young lawyers and senior 

 partners. Often young lawyers are asked to do something of a questionable ethical nature

for a senior partner.

2. Rules 5.1 and 5.2 govern law firms and associations:

a. Under 5.1 - responsibilities of partners, managers, and supervisory lawyers: lays

down the rules for senior partners and supervisory attorneys, requiring that they "keep

house" by providing for rules whereby all lawyers at the firm will abide by the rules

of conduct.

 b. Under 5.2 - responsibilities of a subordinate lawyer: we as individual lawyers are

 bound by the rules. We don't have any excuse for not following the rules because

somebody told us not to follow them in a particular circumstance.

Page 10: Winer Profrespons Fall 2010

8/2/2019 Winer Profrespons Fall 2010

http://slidepdf.com/reader/full/winer-profrespons-fall-2010 10/47

3. Duty to report misconduct under Rule 8.3: this rule concerns a lawyer's obligation to

report conduct of a lawyer that raises questions about his honesty or truthfulness.

V. Corporate Fraud and Lawyer Action

A. General Notes

1. What is "recklessness"?

a. U.S. v. Benjamin: a deliberate act with a mind to doing wrong is not required. A

lawyer simply by closing his eyes to the activities of his client could be behaving

recklessly.

2. Former Model Rule 1.13 - organization as client

a. A lawyer's duty to safeguard a clients interests extends to organizations as a

whole, not just a single person.

i. Means that if we are representing a corporation, the client is the corporation

itself.

ii. A lawyer does not need an exception to the duty of confidentiality in order 

to report constituent misconduct within the organization.

-> 1.13(f) places the burden on the lawyer for the organization for proving

there are a diversity of interests that he represents by explaining the identity of 

his clients.

iii. Confidentiality and 1.13: the client in an organization is the corporation itself 

which is controlled by a board of directors.

-> If the corporation is in receivership, the new "corporation" can vow to have

the past confidential designation removed.

iv. 1.13's trigger for lawyer response: in order to report misconduct by the

organization, a lawyer must actually know something, not merely suspect it in

regards to a matter related to the representation that is a violation of law that

could be imputed to the organization and which could cause significant damage.

v. Whistle-blowing requirement: 1.13(c) is an exception to the rule on

confidentiality 1.6 where a lawyer has reported the activity or the conduct to the

highest authority in the organization, he may disclose details about the

representation to the extent that the lawyer reasonably believes the disclosure is

necessary to prevent substantial injury.

Page 11: Winer Profrespons Fall 2010

8/2/2019 Winer Profrespons Fall 2010

http://slidepdf.com/reader/full/winer-profrespons-fall-2010 11/47

3. A law firm or lawyer should be careful not to have too many eggs in one basket:

the danger highlighted in many of the larger corporate fraud cases is that a law firm

having one client represent a sizable chunk of the firms business (i.e. greater

than 50%) is more likely to be either pushed around by the client or be subject to

the rise and fall of the fortunes of the underlying majority client .

B. Lincoln Savings and Loan

1. The scheme: lawyers for Lincoln were charged with aiding the bank in making

bogus bond sales (common law securities fraud). The law firm essentially offered advice

to ACC (the parent of Lincoln) about how to sell bonds that appeared like they were

 backed by the Feds. Another scheme devised by Lincoln was the land/tax scheme that

had them taking land they had already owned and making it appear like they had made a

 profit on it. The law firm assisted by offering advice for both of these schemes.

2. Rule - Due Diligence: Opinion letter with no follow-up: after conducting a

 background check of ACC's compliance with federal regulations, there were numerous

violations found. The law firm offered advice on how to make violations transparent.

a. Jones day told the ACC to follow compliance advice but there was never any

follow-up.

 b. The problem here is that Jones Day failed to withdraw their representation

when, after finding violations by their client and notifying them, the client failed to

comply and the law firm failed to follow up.

3. Model Rules Implicated:

-> M.R. 1.16: if a lawyer is aware of wrongdoing by his client and, after notifying the

client the wrongdoing continues, the lawyer must withdraw his representation of the

client.

-> M.R. 3.3: although here the SEC may not have been considered a "tribunal" in the

strict sense, the Jones Day may nevertheless have been required to show candor to the

SEC.

-> M.R. 4.1: was Jones Day was engaged in a process of hiding loan documents and

other incriminating materials from federal regulators, which would implicate 4.1

which requires truthfulness in statements to others.

Page 12: Winer Profrespons Fall 2010

8/2/2019 Winer Profrespons Fall 2010

http://slidepdf.com/reader/full/winer-profrespons-fall-2010 12/47

**Note: 4.1(b) contains a reference to rule 1.6 corresponding to a lawyers duty to

disclose material facts to a person when such disclosures are necessary to avoid

assisting criminal or fraudulent activity.

-> M.R. 1.6: although MR 4.1 requires a lawyer to be truthful in his statements to

others, 1.6 also prevents a lawyer from disclosing client confidences. HOWEVER ,

1.6 contains permissive language saying that a lawyer may reveal under certain

circumstances information about his client. One of these circumstances would appear 

to be a case where the lawyer is assisting the client in committing a crime.

C. O'Melveny & Myers

1. O'Melveny was hired by ADSB to author these private placement memoranda for some

real estate developments. The firm knew that ADSB had cooked its books and misstated

its finances, didn't communicate with the accounting firm on file for the bank to verify

finances, and used out-of-date records without consulting other actors in compiling the

memoranda. Investors were mislead by the memoranda in making investment decisions

and the FDIC ultimately brought suit against the law firm.

2. Rule - Due Diligence: Lawyers and law firms are obligated to conduct due

diligence to protect their clients. Here, O'Melveny did not make a "reasonable

investigation to uncover false or misleading material".

3. Rule - Duty is owed to corporation itself (which includes investors): under MR 1.13,

O'Melveny did not just owe a duty to the individual corporate officers who the firm was

working with. Rather, the duty was much broader. The duty here extended to both

investors and the corporation itself .

4. Model Rules Implicated:

-> M.R. 1.13:

-> M.R. 1.2(c): the ability of a lawyer to limit the scope of their representation if the

limitation is reasonable under the circumstances. (goes to the issue of due diligence:

here, if all O'Melveny was doing was authoring the private placement memoranda

and that's what they thought they were doing, they should have made this known to

the client).

D. Student Marketing Case

Page 13: Winer Profrespons Fall 2010

8/2/2019 Winer Profrespons Fall 2010

http://slidepdf.com/reader/full/winer-profrespons-fall-2010 13/47

1. There was a proposed merger between two companies: NSMC and Interstate National

Corporation. Two law firms were hired to facilitate the merger. The law firm authored a

merger agreement and a "comfort letter" that stated that all transactions in connection

with the merger were taken in full compliance with applicable law. Both parties agreed to

the terms of the merger. Interstate did not know that they were relying on outdated

financial documents. The accounting firm for NSMC knew that there were problems

with the financial statements of NSMC and he had trouble drafting the comfort letter that

the two parties had agreed to. Lawyers at the law firm knew about the faulty financial

statements but said nothing. Also, the lawyers gave assurances to Interstate that the non-

conforming financial statement would be brought up to date when Interstate voiced some

concern about the non-conformity of the financial statements to the comfort letter. The

deal went forward despite these deficiencies and with no interference by the lawyers

involved for NSMC.

2. Rule - Secondary liability for the lawyers for aiding and abetting: although there is no

 private suit available for aiding and abetting, the SEC can still bring suit based on this

theory. However, the lawyers involved would only be liable for secondary liability.

3. Rule - inaction on a certain matter may constitute aiding and abetting: where a lawyer

who should have taken some kind of action in order to prevent a certain harmful

result from occurring fails to take that action they could be found liable for aiding and

abetting if their action substantially assisted in the violation. Here, the lawyers for both

sides (the lawyers for Interstate and the lawyers for NSMC) failed to take any action to

stop the closing of the deal.

a. Elements of aiding and abetting:

(1) An individual engaged in some kind of wrongdoing

(2) Alleged aider and abettor had a general knowledge that his role was part of an

overall activity

(3) He knowingly and substantially assisted in the violation

4. Rule - Withdrawing from the representation when client continues to participate in

unethical action: where a lawyer knows of wrongful conduct by their client and where

they have notified their client (in the context of a corporation, the higher ups, the

corporate officers) but the client continues to engage in unlawful conduct, the lawyer

Page 14: Winer Profrespons Fall 2010

8/2/2019 Winer Profrespons Fall 2010

http://slidepdf.com/reader/full/winer-profrespons-fall-2010 14/47

must withdraw from the representation and notify possible effected parties and

concerned law enforcement agencies that he is withdrawing from the representation.

a. However, the lawyer must take care not to divulge confidential client information

in the course of this withdrawal unless the circumstances are such that one of the

exceptions under 1.6(b).

E. SEC and the Regulation of Lawyers

1. §102(e)

a. Rule - a method for the SEC to protect its own processes: In general, this provision

authorizes the SEC to discipline professionals practicing or appearing before it in

order to "protect the integrity of its own processes."

 b. Application

(1) In Re Carter and Johnson

-> Two lawyers who had been representing a company before the SEC were

charged with making false representations to the SEC. The two lawyers had

filed an 8-K form that failed to disclose the particulars of the loan agreement

or the existence of a lease maintenance program. But national was having

some adverse financial consequences and had to use the lease maintenance

 program. This fact was misrepresented to the SEC.

Rule: the lawyer must take more affirmative steps to avoid the notion

that he has co-opted into the scheme of non-disclosure.

(2) In Re Gutfreund 

-> Three lawyers were accused of failing to exercise effective guidance over 

their client, a trader, to respond to misconduct (false bids on treasury

securities).

The SEC wanted the lawyers involved to disclose their clients unlawful

conduct to the SEC but the Bar argued that this would be unethical

behavior because it would compromise the confidentiality of the

attorney/client relationship.

F. Lawyers and Primary Liability for Client Fraud

Page 15: Winer Profrespons Fall 2010

8/2/2019 Winer Profrespons Fall 2010

http://slidepdf.com/reader/full/winer-profrespons-fall-2010 15/47

1. Lawyers can be primarily liable for the fraud of their clients if they have knowledge

that their fraudulent or untruthful representations will be seen by a third-party.

(Central Bank )

2. Rule - Affirmative steps taken by the lawyer = primary liability: A lawyer who (1)

knows that the statement will be relied upon by investors; (2) is aware of the material

misstatement or ommission; (3) played such a substantial role in the creation of the

statement that they could be said to have taken affirmative steps to defraud may be

liable to a third-party. Simplified Rule: where a lawyer elects to speak by authoring or 

co-authoring a document, a duty arises for the lawyer to speak truthfully.

a. In such cases where a lawyer knows they are speaking through their client to

another person (i.e. to an investor), the lawyer is obligated to speak truthfully)

(1) Klein v. Boyd: a lawyer who represented a limited partnership and drafted a

 partnership agreement, a subscription agreement, and a disclosure letter reflecting

the compliance history of the partnership. The lawyer learned (after beginning the

lawyer/client relationship) that the client had a shady financial history. The lawyer 

told the partners to disclose this shady history to investors but the partners

refused. The lawyer continued to issue statements to investors that did not reflect

the partners past financial problems.

-> Here, the lawyers should have spoken truthfully to the investors (but not

necessarily blown the whistle).

(2) Enron Case: a law firm which drafted and reviewed various disclosures that

were materially misleading (misrepresentations about the independence of SPE's

from the parent corporation. V&E did not simply represent the client it

participated in a fraudulent scheme designed to mislead third-party investors.

-> Under the ethics rules the law firm should have notified upper levels of 

management and then withdrawn if the fraudulent behavior continued.

 b. Scrivener's Defense: where a lawyer merely passes along information initiated by

a client but does not make any representations of their own, they cannot be held

 primarily liable for fraud or liable as aider's and abettor's.

3. Model rules and primary and secondary liability

Page 16: Winer Profrespons Fall 2010

8/2/2019 Winer Profrespons Fall 2010

http://slidepdf.com/reader/full/winer-profrespons-fall-2010 16/47

a. M.R. 4.1(a): primary liability - a lawyer acts directly to make a communication or 

take some action.

 b. M.R. 4.1(b): a lawyer assists or somehow indirectly contributes to the fraudulent

conveyance.

VI. Attorney / Client Privilege

A. General Notes

1. Distinction between confidentiality and privilege:

a. Confidentiality

i. Scope: Confidentiality concerns a client's affairs. The duty of confidentiality

 protects nearly all information related to the representation no matter its

source.

ii. Rationale: the rationale for the professional duty of confidentiality arises from

agency law and evidence law. Lawyers are agents for their clients (the principal)

and the agent has a duty to treat information from and about their principals as

confidential to the extent that the principal intends.

 b. Attorney/Client Privilege

i. Scope: protects only information shared between a lawyer and his client.

ii. Rationale for atty/client privilege: because we want a client to communicate

openly with his lawyer in order to receive the most effective representation based

on all the information, the privilege is a way to offer assurances to the client that

the communications he shares with his lawyer will be protected.

B. Three issues with any problem discussing attorney/client privilege

1. What is the scope of the principal or privilege or confidentiality?

2. Once we know the scope, what the exceptions to it?

3. What about the waiver?

Example: how can a company waive the privilege so that the company can cooperate

with the SEC investigation and it turns over certain information which might

otherwise be privileged; can the company waive the privileges to the SEC but still

keep it to themselves as to everyone else in the world?

C. Definition of the Attorney/Client Privilege (restatement)

1. A communication

Page 17: Winer Profrespons Fall 2010

8/2/2019 Winer Profrespons Fall 2010

http://slidepdf.com/reader/full/winer-profrespons-fall-2010 17/47

a. We must be careful though when talking about written documents. They are only

protected if they were prepared for the purpose of obtaining legal advice. Simply

transferring the document to the lawyer will not be enough to trigger the privilege.

 b. Restatement describes a communication as "any expression through which a

 privileged person undertakes to convey information to another privileged person and

any document or other record revealing such expression.

c. Exceptions: Privilege DOES NOT extend to facts or their disclosure; only

communications made by the client to the lawyer for the purpose of obtaining legal

advice.

Privilege DOES NOT extend to the identity of the client (except in cases where

the identity of the client are the only missing pieces in the case).

Privilege EXTENDS after a client's death.

2. Made between privileged persons

a. Again, something that is not well understood. One party to the communication is

the client. The other party need not be a lawyer but could also be agents of either

the client or the lawyer who facilitate communications between them and agents of 

the lawyer who facilitate the representation.

3. In Confidence

a. This simply means that at the time and in the circumstances of the

communication, the communicating person reasonably believes that no one will

learn the content of the communication except the privileged person.

 b. Exceptions: 

i. Communication between inmate and attorney: Where there is communication

 between a lawyer and his in-mate client, the communication is still protected even

where a third party (i.e. a prison official) has heard the communication, but the

 point of the privilege has been dissolved. ALSO, where there is reasonable

suspicion that the inmate would use the communication to facilitate terror, a

 prison official is authorized to listen in to the communication, but both the lawyer 

and the client must be notified.

4. For the purpose of obtaining or providing legal assistance for the client

D. Invoking the Attorney/Client Privilege

Page 18: Winer Profrespons Fall 2010

8/2/2019 Winer Profrespons Fall 2010

http://slidepdf.com/reader/full/winer-profrespons-fall-2010 18/47

1. General Procedural Requirements

a. Client must appear.

 b. Client must testify.

c. Client must invoke the privilege in response to a particular question.

2. Burden of proof is on the client: party wishing to invoke the privilege (client) must

 prove that all the elements of the privilege are present.

3. Claim of privilege asserted for documents: in invoking the privilege for certain

documents or other material sought during discovery, client must identify the information

withheld and provide enough identifying information to permit the adversary to assess

whether the claim of privilege is valid.

i. Court can review documents in-camera to determine if privilege exists: In assessing

whether lawyer has asserted valid privilege, court may review documents in camera.

E. Joint Clients and Cooperating Parties

1. In general: if two or more persons jointly retain a lawyer to represent them in a matter,

communications made by any of the clients to the lawyer on the subject of the joint

representation are privileged against the rest of the world but not against other joint 

clients.

i. Rationale: joint clients intend their communications to be secret from the rest of the

world but not against each other because the fortunes of the clients are, while not

necessarily identical, somewhat similar in the outcome sought.

ii. Common interest exception against disclosures: communications between

 persons with a common interest and their lawyers are protected by the attorney/client

 privilege so long as (1) a common interest on one or more issues exists between

the parties and (2) the communication goes to serving this common interest.

F. Corporations and the Attorney/Client Privilege

1. General rule from Upjohn: a lawyer must have access to all levels of employees and

must have such communications privileged if he is to obtain all of the information that

could be relevant and valuable to the representation of the corporate client.

i. Must be applied on a case-by-case basis; not a general rule on attorney/client 

 privilege for corporations.

Page 19: Winer Profrespons Fall 2010

8/2/2019 Winer Profrespons Fall 2010

http://slidepdf.com/reader/full/winer-profrespons-fall-2010 19/47

ii. Scope of the corporate privilege under Upjohn: A communication between any

agent of the organization and the organization's lawyer concerning a legal matter of 

interest to the organization is privileged if disclosed only to:

(1) Privileged persons

(2) other agents of the organization who reasonably need to know of the

communication in order to act for the organization

iii. Current management of a corporation controls the privilege on behalf of the

corporation. When management is replaced, the successor to the past managing

 board controls the privilege.

-> In a shareholders derivative suit, shareholders may challenge management's

decision to invoke the privilege so that the shareholders may gain access to

otherwise confidential information.

2. Rationale for application to corporations: the rationale for having the attorney/client

 privilege apply to corporations is so that a lawyer may ensure that a corporation is

abiding by the law and the fact that corporations are constantly going to lawyers (more

than individuals would) to find out how to handle certain legal issues.

3. Rules implicated in Upjohn: 

i. Rule 3.4(f): a lawyer may not counsel a person not to engage in communication

with another party unless the person being so counseled is a client .

-> Interesting in Upjohn because there is no language in the rule that discusses

former employees and whether they are considered clients. (the problem could

have arose in Upjohn if the corporate counsel for Upjohn had ordered former 

employees not to turn over the questionnaires that had been prepared).

4. Work product doctrine and privileged communications: The court shall protect against

disclosure of the mental impressions, conclusions, opinions or legal theories of an

attorney or other representative of a party prepared in anticipation of litigation especially

where the communications are privileged. (  see extended rule below  )

G. Government's Attorney Client Privilege

1. In general: although the government was once presumed to enjoy an attorney/client

 privilege that was on-par with that of corporations, recent court decisions have eroded the

Page 20: Winer Profrespons Fall 2010

8/2/2019 Winer Profrespons Fall 2010

http://slidepdf.com/reader/full/winer-profrespons-fall-2010 20/47

scope of the privilege such that now the government's privilege is narrower than that

held by private clients.

i. The rationale for narrowing the governmental privilege is that the public has an

interest in having the wrongdoing of the government exposed.

ii. The interest at stake in Upjohn was private but in the case of a government official,

there is much more at stake (the public interest and the viability of the justice system

itself since the government oversees the justice system).

H. Work Product Doctrine

1. General definition: a party may obtain discovery of material that is otherwise

discoverable and prepared in anticipation of litigation but only if there is a showing of 

(1) substantial need for the material; and (2) the attorney faces an undue hardship in

obtaining the substantial equivalent.

2. Distinction between ordinary work product and mental impressions: material other 

than a lawyer's mental impressions, theories, and opinions is ordinary fact work 

product. Opinion work product is any communication or document that contains the

mental impressions or legal conclusions of the lawyer.

i. Ordinary work product discoverable if: the material is being used for 

impeachment purposes, a witness is unavailable or hostile, there would be undue

delay if a party was not given access to the materials, or where the passage of time

makes the material otherwise inaccessible.

ii. Opinion work product not always protected: the court in Upjohn said that

although opinion work product is not subject to the same showing of necessity and

undue burden as would be required for disclosure of ordinary fact work product, there

are "rare" circumstances in which the material could be discoverable.

3. Work product protection claimable by or for a person on whose behalf the work 

product was prepared.

I. Attorney/Client Privilege and the Fifth Amendment

1. No protection of communications if held in the hands of a lawyer and communications

not privileged by attorney/client privilege: the fifth amendment merely protects a person

from being compelled to testify against themselves. However, in the case that a client has

turned over documents to his lawyer (where the documents were created before the

Page 21: Winer Profrespons Fall 2010

8/2/2019 Winer Profrespons Fall 2010

http://slidepdf.com/reader/full/winer-profrespons-fall-2010 21/47

attorney/client relationship commenced, the fifth amendment does not prevent the

documents from being disclosed because it is the lawyer being compelled to turn over the

documents and not the client himself.

2. Attorney client privilege applies to material given to a lawyer when material was not 

 pre-existing to the attorney client relationship

3. Material protected by the fifth amendment:

(1) Testimonial material the existence is itself significantly incriminating

(2) Testimonial material when its possession by the client is itself significantly

incriminating

(3) "Private papers" of the client

4. Required documents doctrine: documents that a person is required to keep by law

cannot be protected by the fifth amendment.

5. Fifth amendment and protection against the incriminating "production" of materials: an

act of producing a certain document is entitled to Fifth Amendment protection and the

immunity grant extends to the content of the documents. Whether the 5th amendment 

 protection applies turns on the issue of how much effort a person has to exert (in terms

of mental energy) to the lead the government to the incriminating evidence. (Result of 

this rule from Hubbell is that those who commit crimes through documents have broader 

 protection against disclosure of potentially incriminating materials).

6. Subpoena's to lawyers: prosecutors must obtain "prior judicial approval" after an

opportunity for an adversarial hearing before serving a subpoena on a lawyer in a grand

 jury or other criminal proceeding to seek evidence about the lawyer's past or present

clients.

J. Exceptions to the Attorney/Client Privilege

1. A dispute concerning a decedent's disposition of property

2. Client crime or fraud

i. General: we do not want a lawyer helping a client commit a crime or fraud even

though the privilege on communications extends to communications about past

conduct that the client engaged in. If it can be established that a client used the

attorney's services to commit an unlawful act, the communications between the

lawyer and the client will not be protected by the privilege.

Page 22: Winer Profrespons Fall 2010

8/2/2019 Winer Profrespons Fall 2010

http://slidepdf.com/reader/full/winer-profrespons-fall-2010 22/47

ii Applies even if the lawyer does not clear the client's unlawful purpose

iii. Crime-fraud exception may apply even when there is no true crime/fraud

committed:

(1) Lawyer self-protection

(2) Disputes in which a trustee or other fiduciary is charged with a breach of a

fiduciary duty by a beneficiary

(3) Disputes between representative of an organizational client and constituents of 

the organization

iv. Procedural requirements under Chen and Zolin

In order to show that a certain communication between a client and his lawyer is

no longer privileged because of client fraud, there is a two-step process:

(1) Government must make a showing that there is a good faith basis that in

camera review of the materials may reveal evidence to establish the claim

that the crime-fraud exception applies

(2) The materials may be submitted for review to determine if crime/fraud

exception should apply

K. WAIVER 

1. Waiver in general: the attorney-client privilege and duty of confidentiality belong

to the client and the client is, therefore, the only one who can waiver her right to the

 protection provided by both of the privileges. The client may waive, either explicitly

or implicitly, either privilege.

-> Client must have full knowledge: a client MUST have sufficient information

to permit her to understand the legal significance of the waiver.

a. Rule 1.0(e):

 b. Rule 1.6(a):

c. Rule 1.8(b):

-> Lawyer may inadvertently lose the privilege for his client through inaction.

This is a question of agency law because a lawyer is impliedly authorized by his

client to waive the privilege.

2. Types of waiver:

Page 23: Winer Profrespons Fall 2010

8/2/2019 Winer Profrespons Fall 2010

http://slidepdf.com/reader/full/winer-profrespons-fall-2010 23/47

a. Implicit waiver: situation where a client waives the privilege in order for the

lawyer to carry out the representation under 1.6(a).

 b. Selective waiver: there is no selective waiver, period. Once waived, the

privilege cannot be reasserted. A client cannot selectively choose to assert the

 privilege in some instances and waive it in others. A client cannot disclose some

 privileged communications supporting his case, and yet claim the privilege for 

other information (sword/shield use of the privilege)

c. Inadvertent waiver: the traditional rule is that inadvertent disclosure waives

the privilege as effectively as an intentional disclosure. But, new technology

has made this traditional rule more problematic. (i.e. emails, e-discovery, etc.)

Newer rule: even if disclosures were made inadvertently, privilege is still

waived unless there was negligence.

-> 4.4(b) requires lawyers who receive information from inadvertent

disclosures to notify opposing counsel. 

d. Waiver by inaction: the privilege can also be waived without action being

taken by the lawyer or his client. If third party testifies that he relied on an

otherwise privileged document to reach his conclusions, the document is no

longer privileged and the author of the document can be questioned about its

contents.

3. Exceptions to waiver:

a. Disputes between lawyer and client: when the client attacks the lawyer's

conduct, whether in a malpractice suit, a disciplinary charge or other setting,

fairness requires that the lawyer be able to use client communications in

defense.

 b. Reliance on advice of counsel: if one of the parties in litigation offers reliance

on the advice of counsel to support a claim or defense in the litigation, fairness to

the opposing party requires that the privilege be waived with respect to all

communications on related matters.

c. Extrajudicial disclosure to a third party: specific communications to a third-

 party may waive the privilege as to that specific conversation but not

Page 24: Winer Profrespons Fall 2010

8/2/2019 Winer Profrespons Fall 2010

http://slidepdf.com/reader/full/winer-profrespons-fall-2010 24/47

necessarily to the broader subject and other communications to a lawyer on

that subject.

 Rationale: there is no reason why extra-judicial statements should waive

communications on an entire subject matter since statements were not made in

an adversarial setting.

BUT... in an adversarial setting, when a party relies on the advice of counsel

to support a claim or defense, reliance waives the privilege not only with

respect to the specific communication upon which the party relied, but also

with respect to all communications on the same subject matter.

VII. Duty of Confidentiality

A. General Notes

1. Lawyer can refuse to give testimony on certain matters: the duty of confidentiality is

much broader than the attorney/client privilege on communications. The duty of 

confidentiality is rooted in agency law.

2. A lawyer is not allowed to talk about his client's case unless the client gives him

 permission to:

a. Confidentiality principle applies regardless of how the lawyer became aware of 

the information.

 b. Even if the information is widely known, a lawyer must not disclose the

information about his client.

c. Rationale: somewhat similar to the reason for the attorney/client privilege: the

 purpose is to encourage our client's to tell us everything they know about their case.

d. Disclosure of information may be permissible if 1) authorized by client or 2)

required by Rule 1.6.

3. Prohibited Uses of Confidential Information

a. A lawyer cannot use client's confidential information to the client's disadvantage.

 b. A lawyer cannot use a client's confidential information for personal enrichment

(Restatement §60(2).

B. Scope of the Duty

1. Much broader than the attorney/client privilege: the ethical duty of confidentiality

applies to a lawyer whether or not the client has told others the same information that she

Page 25: Winer Profrespons Fall 2010

8/2/2019 Winer Profrespons Fall 2010

http://slidepdf.com/reader/full/winer-profrespons-fall-2010 25/47

told the lawyer. Also, any information relating to the representation of the client is

 protected, no matter who the information came from.

2. Cannot voluntarily disclose information relating to the representation: only by court

order, consent, or by a discretionary exception can a lawyer disclose information about

the representation.

3. Duty of confidentiality survives the death of the client: M.R. 1.9(c)(2) requires that a

former client's confidences must be kept unless another rule requires or permits

disclosure.

C. Exceptions to the Duty of Confidentiality

1. Protection of lawyers threatened by a claim or charge brought by the client or a third

 person ( self-defense exception)

a. Cases where the exception arises:

i. When a client charges a lawyer with wrongdoing in the course of representation

ii. When a lawyer sues the client to enforce some duty owed the lawyer (i.e.

 payment of fees)

iii. When a third person accuses a lawyer of wrongdoing in the course of 

representing a client.

 b. Where a lawyer has a lot to lose or a lot at stake in a particular case, he may

disclose confidential information about a past client: in Meyerhofer v. Empire Marine

 Ins. Co., a lawyer from a firm representing a corporation who was being investigated

for securities fraud left the firm and ( fearing legal action against himself ), disclosed

confidential information about the corporate client to the SEC. The court found that,

given the reputation of the lawyer and what he stood to lose if he allowed the lawsuit

to go forward, he was justified in disclosing the confidential information.

(i) Whether Goldberg should have released the information to the SEC in the

way he did is problematic: The court kind of avoids what is a more sticky and

ethical issue: the lawyers disclosure to the SEC. Even under the model rules up

until 2003, there was no provision to allow for the disclosure of client fraud but

there was the lawyer's self-defense exception. Maybe this opened up an end-run to

allow a client access to confidential client information: they can just sue the

lawyer for the client.

Page 26: Winer Profrespons Fall 2010

8/2/2019 Winer Profrespons Fall 2010

http://slidepdf.com/reader/full/winer-profrespons-fall-2010 26/47

(ii) Goldberg should have gone to the board of directors of the corporation

before making the earlier disclosure to the SEC: although Goldberg may have

 believed that he had a duty under federal law (and as a result of the National 

Student Marketing case), to disclose the information to the SEC without

consulting the board of directors. However, per 1.6, Goldberg should probably

have consulted with the board of directors before disclosing this confidential

information.

c. Lawyer must first believe that disclosures are reasonably necessary under 1.6:

1.6(b) provides that disclosures are limited by the requirement that a lawyer 

reasonably believe the disclosures necessary.

2. Protection of innocent third parties who are being or may be victimized by the client

(disclosure when death or bodily harm could result)

a. Balla v. Gambro: Gambro was in-house counsel for a corporation. He offered legal

advice that a particular shipment of product that the company had received was

defective but the company ignores the advice and attempts to sell the defective

 products on the secondary market. Gambro was fired by the company and reported

his findings to the FDA.

Held: Gambro did not have a claim for retaliatory discharge because Gambro

was required by the ethical rules to disclose his client's fraud. Furthermore, if 

Gambro could sue his client for retaliatory discharge, employers would be

unwilling to share client confidences.

Allowing a lawyer to sue his client for retaliatory discharge would effectively

 shift the burden of abiding by the ethical rules to the client himself making 

the client mitigate damage to the attorney for him not abiding by the ethical 

rules.

Rule 1.6: Gambro should have reported the misconduct of the President up

the corporate ladder before going to the FDA. Under 1.6 and by the duty of 

confidentiality, a lawyer should keep client affairs quiet to the extent that he

 believes reasonably necessary.

 b. Hawkins v. King County - rejecting a general duty to warn about dangerous

clients: Sanders was an attorney for Michael Hawkins and was supposed to secure

Page 27: Winer Profrespons Fall 2010

8/2/2019 Winer Profrespons Fall 2010

http://slidepdf.com/reader/full/winer-profrespons-fall-2010 27/47

the release of Michael but only to have him put in a mental institution. Sanders

did not warn the tribunal about Michael's mental condition (nor was he asked

about Michael's mental condition). On his release, Michael injured his mother and

attempted to commit suicide.

Held: the duty in this case was not akin to the psychologists duty in Tarasoff 

 because, here, the victims knew about Michael's violent tendencies. Also,

Michael gave no indication to Sanders that he was going to attack anyone.

-> Rule 3.3 only requires disclosure of client behavior when danger that

client will commit crime is imminent .

See also M.R. 1.14.

 Subsidiary case on duty of lawyer to warn judge of threats against 

him: the court held in Washington v. Hansen that attorney's, as officers

of the court, have a duty to warn of true threats to harm a judge

made by a client or a third party when the lawyer has a reasonable

belief that such threats are real.

3. Prevention or rectification of fraud on the tribunal

a. When is it appropriate for a lawyer to disclose his client's frauds or misdeeds? This

question is illuminated by the O.P.M. case. A few lessons from that case:

i. There are many different positions on whether lawyers should disclose client

fraud. The ABA's position that you cannot disclose client fraud is at war with

what the law wants.

(a). 1.13 was strengthened by the ABA to do two things: 1) strengthen the

duty to report corporate wrongdoing up the corporate latter; and 2) Allow

disclosure outside the corporation of serious corporate wrongdoing that the

highest authority in the organization refused to rectify or prevent.

ii. The "except" clause in 1.6 and the tension within the ethics rules and with

other law

iii. Kutak commission: split on preventing or rectifying fraud. Rectifying is

second best but preventing fraud is harder to do. The requirement to prevent 

client fraud is almost impossible to do because we have no idea what our client is

going to do before they do it.

Page 28: Winer Profrespons Fall 2010

8/2/2019 Winer Profrespons Fall 2010

http://slidepdf.com/reader/full/winer-profrespons-fall-2010 28/47

iv. Rule 4.1 and the interplay with Rule 1.6.

 b. **Crafting an effective rule for disclosure of client fraud:

First variable: What interests and on what level would we use limiting words to

 protect the interests of our client while making the fraud known?

Second variable: Should disclosure be limited, mandatory, or somewhere in

 between?

Third variable: Should disclosure be permitted, mandatory, or discretionary?

Fourth variable: should we qualify the rule for the case where a lawyer's services

have been used to perpetrate a client's fraud?

Fifth variable: when, if at all, would we qualify what the lawyer can or can't do by

first giving the client the ability to have the lawyer do something?

Sixth variable: when do we make a disclosure under 4.1 in conjunction with Rule

1.6?

D. Confidentiality and Privilege in the Electronic Age

1. Lawyer's duty in general: lawyers have an obligation to take reasonable precautions

to safeguard their clients' confidences.

-> M.R. 1.6, Cmt. 16.

2. Lawyer's should caution their clients about the use of electronic vehicles for 

communication: Lawyers should have a policy for their clients to follow that tells them

when they should keep metadata and when they should delete it. Backup systems are also

expensive (see Zubulake for a 7-factor test).

a. Highlights modern-day ethical concern about meta-data: is it permissible to

"mine" a document to reveal hidden metadata? According to the ABA, it is

permissible for a lawyer to mine a document for hidden metadata even if the

receiving lawyer knows that the communication was sent inadvertently. (On the

 basis of 4.4(b)).

 b. Requires that a lawyer play a larger role: a lawyer representing the client where

significant electronic discovery is anticipated must play more than just an advisory

role. Instead, he must be able to play an active role as an advisor, monitor, and

supervisor.

3. Electronic Discovery

Page 29: Winer Profrespons Fall 2010

8/2/2019 Winer Profrespons Fall 2010

http://slidepdf.com/reader/full/winer-profrespons-fall-2010 29/47

a. FRCP Rule 26 Permits discovery of documents and other tangible things.

 b. Defining the proper scope of electronic discovery: in the Zubulake case, the judge

ruled that it shouldn't be presumed that the cost to produce a document should be born

 by the requesting party. Rather, a court must consider whether data were stored in an

accessible or an inaccessible format. For data that was easy to get to, the burden

falls on the requesting party to pay for the cost of getting the information.

i. The Zubulake 7-Factor Test For Inaccessible Data To Determine if Requesting 

 Party Should Bear Costs

c. Duty to preserve electronically stored information: the duty arises when a party

knows or should have known that it possess evidence that is relevant to pending

litigation or litigation that is reasonably anticipated.

i. SPOLIATION INFERENCE: if evidence is lost or has been destroyed, the jury

will be instructed to infer that the evidence would have been unfavorable to

the party with the duty to preserve it.

VIII. Conflicts of Interest

A. General Notes on Conflict of Interest

1. Two Aspects of Conflicts of Interest Between a Lawyer and Clients:

a. The relationship between the lawyer and client: a client must be comfortable with

with the lawyer. He must be able to trust and confide in the lawyer. So the client

cannot be troubled with nagging doubts about the lawyers worrying about what other 

interests may be occupying the lawyers mind and taking him away from effectively

representing the client.

 b. The representation; what the lawyer does for the client: this is from the perspective

of the lawyer. An easy way to see this is that the client may not even know about

the adverse interests of the lawyer and the diversity of interests he is representing.

2. The four underlying concerns of the concept of conflict of interest:

(1) Trust. This gets into the attorney/client relationship. There cannot be any

antagonisms between the attorney and the client because this would undermine the

trust that the client places in the lawyer.

Page 30: Winer Profrespons Fall 2010

8/2/2019 Winer Profrespons Fall 2010

http://slidepdf.com/reader/full/winer-profrespons-fall-2010 30/47

(2) Commitment. How committed to the cause of the client is the lawyer? Is the

lawyer pulling punches or holding back from representing the client with all of this

 because there is some other kind of interest.

(3) Duty of confidentiality. This is seen as a corollary of loyalty. Is the confidential

information being used against the client or former client?

(4) Appearance of impropriety. It looks bad to both the court and clients if a lawyer 

is being pulled in two different directions by the clients he is representing.

3. Conflicts of interest are legal matters: although the guiding, overarching principles of 

conflicts of interest are embodied in the model rules, the language of the rules comes

out of the common law. Thus, conflicts of interest are legal matters. Courts are NOT

bound by ethical rules.

4. Filing suit to disqualify opposing counsel for a conflict of interest:

(1) Don't sit on your hands; bring the suit at the first opportunity once you know

the facts which you think offer you a basis for bringing such a motion.

(2) Do not speak in generalities about a conflict of interest if you think one exists.

a. N.B. Comment 20 provides that the rules should not be used as procedural

weapons; the rules are a practical tool that lawyers can use to guide their conduct.

B. Model Rules on Conflicts of Interest

1. Rule 1.7: Model Rule 1.7 identifies two different types of concurrent conflicts in

litigation, either of which disqualifies the lawyer from representing both clients:

a. Representations that are directly adverse

i. Example: There is no definition of directly adverse in the model rules but a

good example of this conflict is where a lawyer represents her client's

opponent in a matter while also representing the client.

ii. Requirement against direct adversity of interests goes to the issue of loyalty

between a client and his lawyer. There is a clear breach of trust when a client is

represented by a lawyer who is also representing an opponent; the client may

doubt just how devoted to its causes the lawyer is.

 b. Representations that are materially limiting to the lawyers ability to effectively

represent both clients.

Page 31: Winer Profrespons Fall 2010

8/2/2019 Winer Profrespons Fall 2010

http://slidepdf.com/reader/full/winer-profrespons-fall-2010 31/47

i. Focus is on diverging interests likely to have a harmful impact on the quality

of representation. When a lawyer represents two clients with adverse interests, he

cannot devote his full efforts to either client because o their diverging interests.

ii. Difference between the restatements and model rules on the issue of materially

limiting conflicts: restatements are limited to cases where there is material harm

resulting from a lawyer's adverse representations. Restatement uses "materially

and adversely affected" and requires that there be a "substantial risk" (as opposed

to significant risk). See Rest. § 125, 128, and 135.

c. Who is a current client for the purposes of conflicts of interest?

i. Corporate Family Conflicts - Westinghouse: Comment 34 to Rule 1 says that

corporate family conflicts exist when a lawyer representing a corporation also

represents an affiliate and there is an agreement that the lawyer will not make

adverse representations.

ii. Clients on retainers: in IBM v. Levin, the court found a continuing relationship

 between the client and law firm when there were patterns of repeated retainers.

The law firm could have handled the problem by consulting Rule 1.3,

comment 4 providing that a lawyer should clarify in writing when a certain

relationship has ended so that a client does not mistakenly suppose that the

lawyer is looking after the client's interests when the lawyer is in fact not 

looking after the interests.

d. Curing a simultaneous conflict by resorting to the Hot Potato method: the hot

 potato situation occurs when a law firm drops an existing client (like a hot potato) and

turn him into a former client to take on a new, more lucrative client.

i. In Picker v. Varian, a larger firm merged with a smaller firm and attempted to

drop a client at the smaller firm when a conflict arose with one of the larger firm's

clients. The court held that the firm could not do this. Instead, the firm should

have withdrawn from the case before merging.

ii. Comment 4 to Rule 1.7 provides the answer: if a conflict arises after 

representation has been undertaken, the lawyer ordinarily must withdraw from

the representation.

Page 32: Winer Profrespons Fall 2010

8/2/2019 Winer Profrespons Fall 2010

http://slidepdf.com/reader/full/winer-profrespons-fall-2010 32/47

iii. However, where a conflict is thrust upon the firm by a client where the

lawyer or firm could not have foreseen the conflict, the hot potato doctrine

does not apply and the lawyer could drop the client.

2. Rule 1.8: the basic rule is that a lawyer shall not enter into business transactions with a

client unless the terms are reasonable and in writing and advised of the propriety of 

obtaining independent counsel.

3. Rule 1.9:

4. Rule 1.10: deals with imputed conflicts of interest among lawyers in the same firm.

 All potential conflicts under 1.8 (a) - (i) are imputable to other lawyers in a firm BUT

 personal conflicts are NOT imputable.

a. Basic distinction - loyalty and confidentiality: question of whether one lawyer in

a firm has a conflict of interest in representing a client that was formerly represented

 by another lawyer in the firm turns on the duty of confidentiality.

 b. Large firms complicate issue of imputation: Increasingly, firms are larger in size,

with multiple offices, not just in one city or state. Because of the increasing size of 

the firm the question of imputing conflicts within a firm becomes even more

important. Would we impute conflicts from one lawyer in a firm’s office in Chicago

to another lawyer in another office in another city?

c. The case of the roving lawyer: There is a second real concern that is predicated on

the fact that lawyers today are very mobile. One solution to insulate a lawyer in a firm

is to create a chinese wall around the lawyer who represented a client with a conflict

of interest.

i. There is more leniency with government lawyers. The notion is that we don’t 

want to restrict the kind of talent that the government can attract (the government

needs the option to hire the best talent without the fear that the government

lawyers will be unmarketable after they go work at a private firm.)

**C. Consent and Concurrent Representation Conflicts**

1. Analytical Point: the determination that a conflict of interests exists is only the first

step in the analysis under the model rules. In many situations, lawyers recognizing the

 potential conflict of interest will seek to get client consent to "cure" the conflict. This

raises two separate questions:

Page 33: Winer Profrespons Fall 2010

8/2/2019 Winer Profrespons Fall 2010

http://slidepdf.com/reader/full/winer-profrespons-fall-2010 33/47

(1) Is the conflict one that is "consentable" as opposed to one that cannot be cured by

consent?

(2) Did the client give actual consent?

(3) Also, many times we will need to seek informed consent from past clients to

represent another client on a matter that is the same or substantially related, but we

have to give information about the new representation in order to get consent.

This raises issues of the attorney/client privilege and confidentiality. Explore these

issues in any hypothetical!

2. Rationale: Respect for client consent recognizes the importance of a client's right to

select counsel of her own choosing and, within limits, to select the kind of representation

as between full-blown partisanship and intermediation.

3. Non-consentable concurrent conflicts:

(1) Representations prohibited by law 1.7(b)(2)

(2) Representations where client asserts a claim against a different client 1.7(b)(3)

(3) The risk that the representation will be affected by the conflict. 

4. How consent is given (when consent is effective): a client must always give informed

consent. Whether consent is informed is determined by whether the lawyer has

communicated proper information and offered an explanation about material risks

of concurrent representation. 

a. 1.7 (b)(4) requires written consent. Also, the writing must be submitted in a

reasonable time. The restatements require, under §122(1) that the client or former 

client have reasonably adequate information about the material risks of such

representation.

5. Consent plus rule: See 1.7(b)(1) - a lawyer must that he can effectively represent each

client. With respective to concurrent clients, the lawyer must be introspective; he must

ask himself if he can provide effective representation for each of these clients. This is the

consent+ rule because a lawyer must first make a judgment call as whether he can

effectively represent both clients before asking the client for consent.

6. Advance waiver of conflicts: to avoid disqualifications, firms increasingly employ

 provisions in retainer agreements whereby the client agrees to waive certain future

conflicts should they arise. 

Page 34: Winer Profrespons Fall 2010

8/2/2019 Winer Profrespons Fall 2010

http://slidepdf.com/reader/full/winer-profrespons-fall-2010 34/47

a. 1.7, comment 22 sets requirement for a lawyer who wants to get an advance

waiver from a client: a client must reasonably understand the material risks that

the waiver entails. Whether there is such an understanding depends on the following

factors:

(1) The level of detail concerning the types of future representations that might

arise

(2) The sophistication of the client and the familiarity of the client with the type of 

conflict being waived

(3) The continuity of the lawyer-client relationship

(4) Whether consent is limited to future conflicts unrelated to the subject of the

representation

(5) Whether the client is independently represented by another lawyer when

offering consent

 b. 1.7, Comment 21 allows a client to revoke prior consent based on changed

circumstances. This revocation only applies to representation of the revoking client.

Whether revoking client's revocation disqualifies the lawyer from representing other 

clients depends on the circumstances.

D. Screening to Prevent Conflicts of Interest

1. In general:

a. Definition: Rule 1.0 defines “screened” as the isolation of a lawyer from any

 participation in a matter through the timely imposition of procedures within a firm

that are reasonaly adequate under the circumstances to protect information that the

isolated lawyer is obligated to protect under these Rules or other law.

E. Westinghouse v. Kerr-McGee

1. Facts: Westinghouse, which is being represented by Kirkland, the law firm being

sought be disqualified, is suing some members of API. On the other side, Kirkland, at the

same time, is representing some members of API taking a diametrically opposing position

on a factual issue: is the uranium industry competitive enough?

2. Findings of the court:

a. Didn't matter that API was an agency and not an individual client: Even

though, here, Kirkland on one side of the equation was representing an agency as

Page 35: Winer Profrespons Fall 2010

8/2/2019 Winer Profrespons Fall 2010

http://slidepdf.com/reader/full/winer-profrespons-fall-2010 35/47

opposed to an individual client, the court found that a lawyer, in representing an

agency, is represeint the individual members of that agency.

i. Each of the members thought that the law firm was representing them; this is

how a court should analyze whether an attorney/client relationship existed. Also,

members of API shared confidential information with Kirkland; this is setting

up the "trust" element that was talked about earlier.

 b. The court discusses the chinese wall and how a large firm could undertake to

screen a lawyer from the rest of the firm who could have a potential conflict of 

interest.

i. Here the court found that the chinese wall was ineffective because one of the

lead attorneys did work for API that could have implicated work being done for 

Westinghouse.

ii. The court also declined to buy into the theory that a wall or screen could even

 be implemented effectively to prevent the presumption that actual knowledge of 

one or more lawyers in the firm is imputed to each member of that firm. N.B. This

was a case involving a static lawyer who was not moving amongst law firms. So

rule 1.10(a) would apply.

c. Positional conflicts and Comment 24 to Rule 1.7: when a lawyer takes a legal

 position on behalf of Client A seeking a legal result that is directly contrary to the

legal position the lawyer takes on behalf of client B seeking an opposite legal

result in an unrelated matter, the conflict is a "positional conflict".

i. Comment 24 to Rule 1.7: positional conflicts result if there is a significant risk 

that a lawyer's action on behalf of one client will materially limit lawyer's

effectiveness in representing a client in a different case. 

RARELY THE GROUNDS FOR A DISQUALIFICATION MOTION.

F. Fiandaca v. Cunningham

1. Facts: a legal aid society was representing a group of female prison inmates filing suit

against the state for offering unequal facilities and services to female state prison inmates.

The state offers to settle the claim brought by the female prisoners. The state wanted to

settle by offering an out-of-state facility, but the plaintiffs rejected this. Plaintiffs also

rejected a second alternative of an in-state facility at an existing facility (a mental

Page 36: Winer Profrespons Fall 2010

8/2/2019 Winer Profrespons Fall 2010

http://slidepdf.com/reader/full/winer-profrespons-fall-2010 36/47

facility). But there is a separate lawsuit from the people at the mental facility alleging that

their facilities are discrepant as well. The same counsel who was representing the female

 prison inmates was also representing the patients are the mental institution. The court

initially decided the case on the merits but when the case progressed to the remedies

stage, the court drew the line and found a conflict of interest that 1.7 was designed to

prevent.

2. FIndings of the court:

a. Here, the conflict was clear; a direct violation of 1.7: the conflict between

 NHLA's two clients prevented it from acting as effective counsel.

 b. The state (the opposing party in this case) created the conflict: Who should

have standing to assert a conflict of interest? Here, the opposing party (the state) was

never represented by legal aid but the state, by its offer, created the conflict. Should

they have standing? Notice that the U.S. Supreme Court has been cutting back in a

variety of conflicts about standing. This might be an era that is more susceptible not

to dismiss some of the disqualification notions on the grounds that the party raising

the conflict does not have standing to sue.

i. Procedural issues involved in raising a conflict of interest: do non-parties

unaffected by a particular conflict of interest (in the sense that the party is not

represented by a lawyer undertaking multiple, conflicting representations) have

standing to bring suit based on conflict of interest? This used to be a comment 

(1.7[15]) but it was removed. Courts now require some kind of personal stake in

the issue or a "concrete" and "particularized" interest.

c. Legal service organizations can have conflicts of each individual member

attributed to the whole organization: It is somewhat of a slippery slope. Are we

going to say that public defenders don’t have to live up to the standards of other 

lawyers? There is a real problem if we say that lawyers don’t have to take account of 

conflicts of interest. But the argument is that if you are not going to cut the slack for 

large law firms, why should you cut the slack for legal defense organizations?

i. Also, there are implications for access to justice in limiting the activities of 

legal service organizations. A legal aid firm may undertake concurrent

representation of co-defendants (a Cuyler problem) but could be disqualified from

Page 37: Winer Profrespons Fall 2010

8/2/2019 Winer Profrespons Fall 2010

http://slidepdf.com/reader/full/winer-profrespons-fall-2010 37/47

representing both of the clients if the representations are directly adverse or if the

lawyer would be materially limited in either of the representations.

d. Appealability of a disqualification: If you have the basis for a good

disqualification motion but you don’t say anything to the court, you may lose the

ability to disqualify. If you want to bring a disqualification motion, make sure you

bring it as soon as you find out about it.

G. Remedies for Conflicts of Interest

1. Malpractice suit filed by the affected client. 

2. Fee forfeiture by the lawyer: a lawyer cannot collect after conflict arose.

3. Disqualification of lawyer with conflict. This is the most common remedy.

H. Concurrent Representation in Criminal Litigation

1. In general: Here, there is a great danger that there could be a simultaneous conflict

 because co-defendants are often represented by a single lawyer. The problem is that there

is a temptation by one client to point the finger at another client.

2. Cuyler v. Sullivan

a. Facts: Cuyler and others were convicted of murder. Sullivan, initially couldn’t

afford counsel so he accepted representation by a lawyer representing the other co-

defendants. Sullivan is convicted and the two other defendants are acquitted at their 

separate, later trials. At no time did Sullivan or his lawyer object to the multiple

representation. Sullivan sought state collateral review arguing ineffective assistance

of counsel. Notice that often that claim is that the lawyer is “asleep at the wheel”. The

claim here is somewhat different in that it says that the lawyer is ineffective because

of the conflict.

 b. Findings of the court:

i. Not the court's responsibility to seek out potential conflicts between a

lawyer and his client: lawyers have an ethical obligation to avoid conflicting

representations and to advise the court promptly when a conflict of interest arises

during the course of a trial.

--> Attorney's themselves are in the best position and it is in their best

interest to determine when a conflict exists.

Page 38: Winer Profrespons Fall 2010

8/2/2019 Winer Profrespons Fall 2010

http://slidepdf.com/reader/full/winer-profrespons-fall-2010 38/47

--> A court can only inquire into a potential conflict after an objection is

made by the lawyer or the client .

--> Rationale is similar to that of the court's in Balla v. Gambro where the

court found that it is permissible for the court to rely on the professional

responsibility of lawyers to identify and rectify conflicts (self-regulation in

a sense).

ii. Ineffective assistance of counsel resulting from a conflict of interest: Even

though there are conflicts in every instance (potentially), the court will not

 presume from the mere fact of multiple representation that there was ineffective

assistance. Instead, in order to prevail on an ineffective assistance of counsel

claim, the client will have to demonstrate that there was an actual conflict 

which actually impacted the lawyer's ability to render effective representation.

(a client need not demonstrate prejudice as a result of the conflict in order to

obtain relief on the theory of ineffective assistance of counsel).

--> TEST: in order to establish a violation of a constitutional right to effective

assistance of counsel, defendant wishing to maintain the claim must

demonstrate an actual conflict of interest that adversely affected the

representation.

--> TEST (OUTLINED): A defendant, in order to show ineffective

assistance of counsel must:

1. Make a timely objection before trial: defendant has to show that the

 potential conflict is such that it is likely that the representation will be

materially impacted by the conflict and would prejudice the lawyers

ability to make an adequate representation

2. If no objection before trial: defendant raising the objection is looking

retroactively and he must show an actual, adverse impact on the

representation.

3. Waiver and Denial of Defense Counsel of Choice: criminals defendants are often,

though not always, allowed to waive conflicts of interest under the ethics rules, but

should they be able to do this?

Page 39: Winer Profrespons Fall 2010

8/2/2019 Winer Profrespons Fall 2010

http://slidepdf.com/reader/full/winer-profrespons-fall-2010 39/47

a. Constitutional implications: when a defendant is denied counsel of her choice, the

defendant's sixth amendment right to counsel may be violated. It isn't as clear 

whether, if a defendant is denied their choice of counsel if they must show 

 PREJUDICE in order to have a constitutional claim.

 b. In Wheat v. U.S. the court said that district courts have considerable discretion

in denying a defendant’s waiver of any conflict even if this denial of waiver 

inhibits the right of a defendant to choose his own lawyer.

i. Rationale: pre-trial conflicts are very hard to predict because of the nature of 

the proceedings before a trial. This seems to be a case against multiple

representations in an implicit sense.

c. Procedural requirements: in federal cases when one lawyer proposes to represent

co-defendants concurrently, the judge must hold a hearing to advise each defendant 

of her right to separate counsel. Federal Rules of Criminal Procedure 44(c).

I. Joint Representation in Transactions and Civil Litigation

1. In general: Here the calculus is somewhat different because we don’t have the

constitutional concern. What is the argument for allowing multiple representation here?

The main argument is cost.

2. Model Rule 1.7 is applicable to joint representations in civil trials: 1.7 is broadly

 permissive of joint representations so long as clients given consent.

a. Rule on informed consent: As long as there is informed consent, these kinds of 

multiple representations are generally allowable by the rules. Look at the Tom, Dick,

and Harriet hypothetical. This hypothetical was posed to very highly regarded

 business lawyers who reached opposite conclusions. One lawyer said there was no

ethical problem, while the other lawyer says that he could see some pessimistic

 possibilities down the road. Rule: informed consent must set the ground rules

before a lawyer undertakes multiple representations in a civil case.

 b. State v. Callahan

i. Facts: Respondent, attorney John Callahan, was acting as personal attorney to

Lowell Lygrisse. Lygrisse was looking to buy land owned by California resident

Ruth Fulton, and suggested to her that Respondent handle the transaction for both

of them. She agreed, and Respondent successfully drew the appropriate papers

Page 40: Winer Profrespons Fall 2010

8/2/2019 Winer Profrespons Fall 2010

http://slidepdf.com/reader/full/winer-profrespons-fall-2010 40/47

(including a very unusual arrangement that included an unsecured, interest-free

loan as part of the purchase price that Fulton did not fully understand) and closed

the sale as per prearranged terms. When Lygrisse defaulted on the final payment,

Fulton came to the Respondent for advice. Respondent repeatedly advised Fulton

that Lygrisse would pay. When he did not, she retained separate counsel for a

malpractice action against Respondent once it became clear that the sale

agreement had given them no security interest in the property. Respondent argues

that he was merely acting as a “scrivener” for the two parties, and that he had no

obligation to suggest “better terms” than t

hose provided by Lygrisse.

ii. Findings of the court:

Rule 1.7: A lawyer must consider that his judgment could be impaired in a

case of representation of multiple clients.

Obligation of lawyers to ensure client understanding: Lawyer’s are

obligated to make sure the client understands A) the nature of the relationship;

and B) the nature of the work being done for the client.

iii. Other rules implicated:

Rule 2.4 - lawyer serving as third-party neutral: this rule uses the term of 

"third party neutral" which could be a lawyer serving as an intermediary as a

mediator, arbitrator, etc. If lawyers are to serve as a third-party mediator, they

must be neutral (including freedom from conflicts of interest)

--> Comment 3 of 2.4 states the notion of confidentiality and the

inapplicability of the attorney/client privilege: there is tremendous

 potential for confusion among the parties if a lawyer serves as an

intermediary (because the parties may believe that they lawyer is acting on

their own behalf). Consequently, a lawyer must inform unrepresented

parties that the lawyer is not representing them.

Rule 1.12 - lawyers who were former arbitrators shall not act as counsel

to any party involved in the mediation: lawyers shall not represent anyone

in connection with a matter in which the lawyer participated personally and

substantially unless all parties give informed consent. If a lawyer wants to

Page 41: Winer Profrespons Fall 2010

8/2/2019 Winer Profrespons Fall 2010

http://slidepdf.com/reader/full/winer-profrespons-fall-2010 41/47

represent one of the parties from the mediation, there must be informed

consent from all of the parties. 

3. Aggregate Settlement Rule: Rule 1.8(g) provides that a lawyer participating in the

aggregate of clients' claims must obtain the informed consent of each client in a

writing signed by the client.

a. Comment 13 to 1.8 provides that lawyer should disclose what the other clients will

receive or pay (this raise 1.6 concerns)

 b. This rule could also tempt a plaintiff's lawyer to push for global relief which is

somewhat anonymous and could short-change a group of litigants (particularly

defendants).

J. Successive Representation

1. In general: As the book says, it is kind of like a case of “switching sides” where a

lawyer represents on client in a matter and then, essentially, switches sides. These

representations are fully consentable. All a lawyer needs is informed consent. The

question is informed consent from whom. The rule itself talks about informed consent

from the former client:

a. We may go to our former client and get consent. But, we may have to disclose

information about our current client. We would then have to get informed consent

from our current client.

 b. The current client might not like the fact that a former client was represented by a

lawyer with an interest that is adverse to the client being represented currently.

2. 1.9 rule on successive representations - former clients: this is the substantial

relationship test. A lawyer who has formerly represented a client in a matter shall not

represent another person in the same or a substantially related matter in which that

 person's interests are materially adverse to the interests of the former client unless the

former client gives informed consent.

a. Consent must be in writing.

 b. Consent not explicitly required. Rather, need for consent may be created by Rule

1.7. A lawyer's duty to a former client could give rise to a conflict with a current

client.

Page 42: Winer Profrespons Fall 2010

8/2/2019 Winer Profrespons Fall 2010

http://slidepdf.com/reader/full/winer-profrespons-fall-2010 42/47

c. Under 1.9(c), consent from a former client that a lawyer may take on a particular 

representation does not extend to a waiver of a lawyer's duty of confidentiality to

that former client.

3. Successive Representation of Joint Clients:

a. Brennan's Case

i. Facts: Wegmann was the lawyer for the defendants and he hired another lawyer,

Sprung, to assist in the matter. However, the plaintiffs (the Wegmann family)

moved to have Wegmann disqualified on the basis of conflict of interest.

Wegmann had previously applied for trademark applications for the Brennan

family corresponding to the Brennan's family restaurant name. However, there

was division in the company and Brennan's restaurants split from the family

 business taking the Brennan's restaurant trademark with them. In a trademark 

infringement suit, the Brennan family sought Wegmann's disqualification as

counsel for Brennan's restaurants on the ground that his present representation

was at odds with the interests of the plaintiff, his former client.

 Notice this is a classic case of an attorney following his own nest. If the attorney

who drafts his will on behalf of the testator and now a possible beneficiary wants

to hire that lawyer to attack the validity of the will with respect to their omission.

We are asking the lawyer to attack his own work product in effect. This is a

 particularly egregious type of conflict.

ii. Findings of the court:

(1) No breach of confidentiality here - joint representation: Because there

was no confidentiality as between the two different clients (because they were

 joint clients), all the court could rule on, at least as grounds for a conflict of 

interest, was the appearance of impropriety, a breach of the plaintiffs'

loyalty and trust.

(2) Wegmann disqualified because the ethical duty to maintain a client's

confidences is broader than the evidentiary privilege: the obligation of an

attorney not to use information acquired in the course of representation serves

to vindicate the trust and reliance that clients place in their attorneys.

Page 43: Winer Profrespons Fall 2010

8/2/2019 Winer Profrespons Fall 2010

http://slidepdf.com/reader/full/winer-profrespons-fall-2010 43/47

 b. Other successive representation issues

i. Substantially Related Matters

(1) Courts have employed different formulas for determining what

"substantially related" means. Rule for the course: Substantially related

means that there is a risk of divulgence of confidential information. The

courts go along something of a continuum: some have a notion that “till death

do us part” while others have the notion of “here today, gone tomorrow”.

-> Comment 3 to 1.9 defines matters as "substantially related" "if they

involve the same transaction or legal dispute or if there otherwise is a

substantial risk that confidential factual information as would normally

have been obtained in the prior representation would materially advance

the client's position in the subsequent matter.

(2) Notion of "playbooking": This issue will often come up when we are

representing an entity. Lets say we are representing our client and over a

 period of time we get to know the people are the corporation-client and we get

to know their attitudes on risk, negotiation, settlement tactics. A new client

comes along and wants to sue our client. Arguably, we have a good idea about

how our current client will play the game. This is where “playbook”

information comes into play. Playbook information is information about our 

client and how they play the “game”. Ordinarily, the playbook information

isn’t considered a basis for disqualification. Professor doesn’t agree with this

because professor believes that with this playbook knowledge lawyer for an

organization could have an unfair advantage.

--> Comment 6 to Rule 1.7 discusses the notion of representation of 

"economically adverse" clients saying that it is OKAY for a lawyer to

represent entities which compete economically so long as the

representation is undertaken in an unrelated action.

K. Imputed Conflicts and the Migratory Lawyer: the applicable rule here is 1.10. Unless it is

a personal conflict between a lawyer and a client, Rule 1.10 is very clear in its meaning. This

rule is very easy to apply, but it also has the beneficial effect creates the problem of lawyers

Page 44: Winer Profrespons Fall 2010

8/2/2019 Winer Profrespons Fall 2010

http://slidepdf.com/reader/full/winer-profrespons-fall-2010 44/47

carrying with them all kinds of “disease” of communicable conflict that could affect other 

lawyers in practice with them.

1. Three questions with moving lawyers and conflicts of interest:

a. What can the individual lawyer do?

 b. What can the new firm that the lawyer joined do or not do? (1.10(a))

c. What can the old firm that the lawyer left do or not do? (1.10(b))

2. Imputed Disqualification and Migratory Lawyers:

a. Nemours Foundation v. Gilbane

i. Facts: Bradley assisted a lawyer at his first firm in preparing work for a “mini-

trial” representing Furlow. Bradley claimed that he couldn’t really remember 

anything from this first litigation. (This brings up a great question about how

much should a lawyer be expected to forget about things from a previous case?)

What helps in this case is that Bradley says that he doesn’t remember anything

despite his significant involvement. (There is a lesson here: we should keep good

records about what we did in a our representation of a particular client).

Bradley then moved to a new firm a few months later. In October 1985, Nemours

moved to disqualify Biggs. But it is quite clear from the first few words of the

opinion that Biggs will not be disqualified.

Disqualification of Bradley is pretty easy because he worked at a firm with a

client that is now a party in an issue that Biggs is representing the company on.

 Notice here that although Bradley did not directly represent Nemours in the first

litigation, Nemours nevertheless provided some information to Bradley to the

 point that there was enough for him to be qualified. (See 1.10(a)).

Bradley is knocked out at Bigg’s firm so shouldn’t the firm be knocked out too?

 NO, the court says. Biggs argues that there was an effective screen built around

Bradley. The court picked up on Biggs’ argument that while the rules did not

really say that screening was not allowed, the drafters could have intended for 

Page 45: Winer Profrespons Fall 2010

8/2/2019 Winer Profrespons Fall 2010

http://slidepdf.com/reader/full/winer-profrespons-fall-2010 45/47

screening to be permissible because there was nothing in the rules directly

 prohibiting screening. Secondly, Bradley imposed his own cone of silence. 

ii. Findings of the court:

(1) In constructing a screen to prevent imputation of conflicts on to other

lawyers, an individual lawyer should take the initiative to put up the

screen: the court is focused on the individual lawyer and this is good because

Bradley is the person who has the confidential information (obligation of 

 screening begins with the individual lawyer and the firm itself has an

obligation to support this individual screen with a screen of their own).

(2) The court balanced the effect of disqualifying Biggs as counsel for the

particular client against the danger that there would be a conflict of 

interest. On balance, Biggs had instituted effective measures to ensure that no

conflicts of interest would arise and, because Biggs was only one a few firms

in this area combined with the fact that an excellent working relationship

 between the client and the firm had arose, this was enough to avoid

disqualification.

XI. Competence and Ineffective Assistance of Counsel

A. In general: competence is not just about intelligence or capability but is also about caring

in a variety of respects.

B. Ethics rules on competence:

1.1 - competence: a lawyer must be competent in his practice and can either have the

knowledge of a general practitioner or a specialist depending on the demand.

1.3 - diligence: a lawyer should be a zealous advocate for their clients.

1.4 - communication: five specific aspects of a lawyer's duty to communicate with his

client.

C. Effective Assistance of Counsel and the 6th Amendment:

1. Current 6th amendment rule: counsel must be appointed in every criminal case,

including misdemeanors, in which a prison sentence may be imposed, unless the

defendant exercises a right to self-representation.

2. When is appointed counsel competent under the 6th Amendment:

a. Strickland v. Washington:

Page 46: Winer Profrespons Fall 2010

8/2/2019 Winer Profrespons Fall 2010

http://slidepdf.com/reader/full/winer-profrespons-fall-2010 46/47

i. Facts: Strickland (D) was charged with murder, robbery, kidnapping, torture,

and various other felonies. D pleaded guilty against his counsel's advice, waived a

 jury trial and sentencing, and chose to be sentenced by the judge. Counsel decided

not to present evidence of D's mental state because he believed that the

 prosecution would be able to impeach any such evidence. D was sentenced to

death. D sought habeas corpus claiming ineffective assistance of counsel. The

Florida Supreme Court affirmed. A U.S. district court denied habeas corpus, but

the court of appeals reversed. The Supreme Court granted certiorari.

ii. Ruling of the court:

(1) Objective standard for determining competence: the standard for 

competence that the appellate court cited the notion of reasonableness:

“Counsel reasonably likely to render and rendering reasonably effective

assistance given the totality of circumstances.”

(2) Court fails to define what "reasonableness" is: claimed that standards

for conduct and competence were set by the ABA and these are a baseline for 

determining whether counsel's conduct was reasonable.

--> Gives high degree of deference to lawyers and the bar to determine

what competent lawyering is. Lawyers are already bound by the ethical

rules and they have an incentive to act according to the rules.

(3) Constitutional standard for determining competence: the overarching

standard is whether the competence was such that it would bring a fair trial

and a just result. Requires a showing that a client's lawyer's acts or omissions

were "outside the wide range of professionally competence assistance." In

order to prove ineffective assistance of counsel, a defendant must show two

things:

(A) Counsel's performance was deficient.

(B) The deficient performance prejudiced the defense.

**Requires a showing that counsel's errors were so serious as to deprive

the defendant of a fair trial.

X. Client Perjury

 

Page 47: Winer Profrespons Fall 2010

8/2/2019 Winer Profrespons Fall 2010

http://slidepdf.com/reader/full/winer-profrespons-fall-2010 47/47